« first day (1670 days earlier)      last day (3352 days later) » 

3:01 PM
Hi
According to http://dlmf.nist.gov/13.3.E22 the first derivative of the confluent hypergeometric function can be wrriten as:

$$\frac{d}{dz}\mathop{U}\nolimits\!\left(a,b,z\right)=-a\mathop{U}\nolimits%
\!\left(a+1,b+1,z\right),$$

my question is that; if the instead of $z$ we have $z^{-1}$, what will be the difference?
 
@TedShifrin To come back to the previous exercise, how do you get from $X=P\begin{bmatrix}0&a&b\\0&c&d\\0&0&e\end{bmatrix}P^{-1}$ that the space is two dimensional ?
 
Huh, no @Hippa. You'll get a matrix of the form $\begin{bmatrix} 0 & 0 & 0\\ 0 & a & -b \\ 0 & b & a \end{bmatrix}$.
 
@TedShifrin Oh true, it can be done that way too, I had forgotten :D
I just trigonalized it
 
@TedShifrin I think your counterexample can't disprove: 3^{n} \equiv 3^{n \mod k} (\mod k) I tried many examples and it really seems to be true
 
@DanielFischer I have seen that it doesn't give the right result. Now I tried the following, that gives the right result?
int ipow(int base, int exp) {
    int result = 1;
    while (exp!=0) {
             result*=base;
             exp-=1;
    }
    return result;
}
Is it the most effective? Or could I improve something?
 
3:05 PM
$k=3$, $n=4$ was my counterexample @Silettiffy
 
@TedShifrin Urm so how do we link $A$ to the solutions of $AX'=X$ ?
 
Change basis to make $A$ in the form I just typed, @Hippa.
 
@Chris'ssis: Does Catalan come up in that integral. A lot of approaches have parts that include Catalan, but perhaps it gets cancelled out by the other parts that I haven't gotten yet
 
$k=3$ and $n=4$ is not a counterexample for $3^4 mod 3 = 3^1 mod 3$
 
Today I went out for a walk and a haircut. I feel much better now.
 
3:09 PM
hi @robjohn
hi Jasper ! Good :)
 
@evinda For exp >= 0, while you don't have overflow, it gives the right result. It's the naive O(exp) algorithm, by a far cry not the most efficient.
 
@TedShifrin hey there... we are in the middle of a heavy rain. We really need it.
 
yes, I know ... I would rather not move to a fire zone.
 
I'm considering the interval $(0, 1)$ on $\mathbb{R}$ with the zariski topology. Under this topology, would I be correct in saying that it isn't closed as it neither has finitely many points nor is it all of $\mathbb{R}$? And hence it is not compact by the Heine-Borel theorem?
 
@TedShifrin $\begin{bmatrix}ae^{bt}\\ce^{dt}\\fe^{gt}\end{bmatrix}$ defines a plane ?
 
3:11 PM
@TedShifrin fires were not too bad last summer. It is the drought that hurts.
 
no, @Hippa. You're not doing this right at all.
 
:/
AX'=X would give us something of the form $\begin{bmatrix}x'\\y'\\z'\end{bmatrix}=\begin{bmatrix}0\\\alpha\\\beta\end{bmatrix}$
 
@Hippalectryon definitely not, unless $b=d=g$ and then it would be a line
 
@Hippa: You're confuzled.
 
@Hippalectryon is $A$ a constant matrix?
 
3:14 PM
yeah, @robjohn. He's looking at the usual $X'=AX$ ...
 
@robjohn yes, with $\det A=0$
 
with $\det A = 0$
 
@TedShifrin nice problem
 
and similarly, $\{0, 1\}$ would be a compact subset under this topolgy, wouldn't it?
 
I thought you were looking at the open interval, @user112495
 
3:16 PM
wrong delimiters, I think
 
@TedShifrin These are two different questions.
@TedShifrin I just want to see if I'm going about these the right way.
 
@user112495 If those delimiters are correct, that would be compact under any topology
 
@user112495: Any topological space is itself both open and closed.
 
@TedShifrin Urm I'd be most confused indeed if we didn't get $\begin{bmatrix}x'\\y'\\z'\end{bmatrix}=\begin{bmatrix}0\\\alpha y\\\beta z\end{bmatrix}$
 
No, @Hippa, not in the complex eigenvalue case. In the real eigenvalue case, yes.
No, no, either way.
You're missing $\alpha y$, $\beta z$.
 
3:18 PM
I just did $\begin{bmatrix} 0 & 0 & 0\\ 0 & a & -b \\ 0 & b & a \end{bmatrix}\begin{bmatrix}x'\\y'\\z'\end{bmatrix}$
ooh
Typo q_q
 
@TedShifrin @robjohn I'm looking at the Zariski topology on the set $\mathbb{R}$. I'm then looking at the subsets $(0, 1)$ and $\{0, 1\}$ to determine whether or not they're compact and/ or closed.
 
Ohhh, I misunderstood the first one, @user112495, sorry.
The second one is both. The first one is neither.
 
@TedShifrin So, the corrected one is ok ?
 
yes, @Hippa.
Now think about solving it.
 
@TedShifrin Are the reasons I gave above the correct way of getting to these answers?
 
3:20 PM
@TedShifrin Doesn't that give us the exponentials I wrote above ? @TedShifrin
$\begin{bmatrix}a\\ce^{dt}\\fe^{gt}\end{bmatrix}$
 
hell no, @Hippa :P
 
Well $\begin{bmatrix}x'\\y'\\z'\end{bmatrix}=\begin{bmatrix}0\\\alpha y\\\beta z\end{bmatrix}\Rightarrow\begin{bmatrix}x\\y\\z\end{bmatrix}=\begin{bmatrix}c\\ne^{‌​\alpha t}\\me^{\beta t}\end{bmatrix}$
 
yes, @Hippa, in the case that $\alpha$ and $\beta$ are real. ... No, you have your constants of integration wrong.
 
Oh yeah wrong ones
 
@DanielFischer We could use the following algorithm, right?

Alg(int n, int k){
    if (k==0) return 1;
    result=Alg(n, floor(k/2));
    result=result*result;
    if (k==1 mod 2) result=result*n;
    return result;
}
@DanielFischer Its time complexity is $T(k)=T(floor(k/2))+d$
a=1, b=2, f(n)=d
$n^{\log_b a}=1$
So $f(n)=\Theta(n^{\log_b a})$, so from the second case of the Master theorem, we have that $T(k)=\Theta(\lg k)$, right?
 
3:24 PM
@evinda Have you followed the code to understand it and implemented and tested to check that it apparently works?
 
Think about solving $u'=\lambda u$ when $\lambda = a+bi\in\Bbb C$, @Hippa.
 
@evinda Yes, complexity is $O(\log k)$.
 
@Ted!
 
hi @Balarka
 
Hi @BalarkaSen
Hi professor @TedShifrin
 
3:25 PM
@user112495 I wouldn't use Heine-Borel....
 
hi @Sayan
 
@DavidWheeler really? But doesn't the result follow straight away from it? Or am I using it incorrectly?
 
I would argue this way: suppose $\mathscr{O}$ is an open cover of (0,1), pick any element of $\mathscr{O}$ that has a non-null intersection with (0,1)
 
@TedShifrin But why are complex numbers involved here ? $A,X$ are real matrices. We got rid of the complex eigenvals by using the modified $A$ matrix.
 
Said set misses at most finitely many elements of (0,1)
 
3:28 PM
because we were talking about the complex eigenvalue case, @Hippa.
You have to solve $y'=ay-bz$, $z'=by+az$.
 
So pick one set from $\mathscr{O}$ that covers each of those points-Voila! a finite open subcover.
 
@BalarkaSen how will I write the proof of something like this....
If $A-B={\nullset}$
A is a subset of B.....
I know the idea but how do I write the proof
 
A - B is the collection of all elements in A that is not in B
 
$A - B = \emptyset$ means there is no element of $B$ that is NOT an element of $A$.
 
if A is a subset of B, then every element of A is in B
so there you go.
 
3:32 PM
@DavidWheeler Sorry, could you explain why it misses at most finitely many elements of (0, 1)?
 
@user112495 Zariski topology
 
@DavidWheeler Oh yeah, :p. Thanks!
 
@TedShifrin That'd give us like $y=e^{at}(m\cos(bt)+n\sin(bt))$
 
OK, @Hippa, ça marche.
 
@TedShifrin I have worked out a few exercises of Hatcher and I'll finish the rest of them after I go through chapter 2.1 and 2.2 again in detail to acquire a few geometric ideas.
 
3:33 PM
And... that defines a plane ? (with z)
 
@SayanChattopadhyay do it yourself.
 
yes, @Hippa
 
Suppose $x \in A$. Either $x \in B$, or $x \not\in B$
 
@TedShifrin Ok thanks
 
Fine @BalarkaSen
 
3:34 PM
If $x \not \in B$, then $x \in A - B$, contradiction. Hence $x \in B$.
 
@David! glares
 
has no idea what is going on
 
I know, the law of excluded middle isn't accepted by everyone. But let's pretend, OK?
 
:P
@TedShifrin Oh, and did I tell you that I finally understood the snake map?
 
geometrically, you mean, @Balarka?
 
3:36 PM
yup
 
yippee
 
For noisy abelian categories we have the rattlesnake lemma
 
it was after a bit of help from prof though. i told mike already, let me find the discussion.
 
if i remember the picture you gave, balarka, it (roughly speaking) amounted to viewing a sphere with handles from the 'perspective' of one of the handles?
 
3:38 PM
I'll look later, @Balarka
 
okay.
essentially it sends a relative cycle of H_n(X, A) to the boundary of it, which is a cycle of H_{n-1}(A)
@Semiclassical i don't know what you mean.
 
well, a sphere with handles = a torus of some genus
 
yes, @Balarka ... now do Mayer-Vietoris :)
 
Under the same topology, $\{0, 1\}$ isn't connected is it? As we can decompose it into $A=\{0\}, B=\{1\}$
 
yeah, essentially you look at the genus g surface as g torii squashed
 
3:40 PM
though i guess that analogy doesn't work quite right, since that boundary has the handle 'inside' it
 
then look at the last punctured torus you get by chopping the end off
 
wait, they they aren't closed. Ignore that
 
right.
 
a relative cycle all over the first piece is sent by the snake map inside the boundary of the disc removed from \Sigma_g-1, i.e, the boundary of the first piece
this is a cycle in the second piece
but in turn is a boundary, so has homology class 0
 
i guess the analogy i have mentally in mind is: take a coffee mug, and draw a circle around the handle (not along the loop, but so as to enclose the handle)
 
3:41 PM
yeah. that's right.
@TedShifrin i "believe" mayer-vietoris. it's essentially a long version of van kampen
but i'll read the proof.
 
i imagine you worked out how to make that geometry-story more precise. i haven't worked with homology stuff lately, and when i did it was hardly at a rigorous level
 
hmm. $\mathbb{R}$ isn't a metric space in this case either.
 
no pushouts of non-abelian groups, @Balarka
 
Can I have a hint for showing this?
 
homologically, it all comes down to a short exact sequence of complexes, just like in the relative case
 
3:43 PM
yeah, well, i haven't read it.
 
@DanielFischer At each call of Alg(j,i) we calculate the value $j^{\floor{\frac{i}{2}}}$.
When $j=n, i=k$ we have the desired result.
Right?
 
@user112495 in which case?
 
(which in retrospect is silly---understanding relative homology groups would probably sharpen my understanding of picard-lefschetz transformations of cycles on riemann surface)
 
zariski topology? it's not hausdorff
 
or you can think about integrals and periods, @Semiclassical
 
3:44 PM
that's what i've mostly done, yeah
 
definitely, @Semiclassical. it's frustrating at first because of the huge amount of algebra, but gets very interesting when you see the topology.
 
and monodromy etc
 
@BalarkaSen The set $\mathbb{R}$ equipped with the zariski topology. I'm trying to determine whether or not $\{0, 1\}$ is a connected subset of $\mathbb{R}$
 
@evinda Can you prove it?
 
yeah, it's hard to separate points, because neighborhoods are "big"
 
3:45 PM
nod. knowing a bit about algebraic topology from the level of concepts is a useful thing
 
@Semiclassical well, you mean an algebraic proof (because that geometry-story is essentially quite precise)?
@user112495 zariski topology is not metrizable. hausdorffness gets void.
open sets are huge.
 
@Balarka still hates non-Hausdorff :D
 
no. i just mean that i haven't heard about the snake map till now, so i'm not in a position to 'truly' appreciate the geometric picture you gave
 
@DanielFischer Should I prove it using the invariant that result is at each step of the form $j^{\floor{\frac{i}{2}}}$ ?
 
it is T1, though, so it gets good download speeds.
 
3:46 PM
i can see the intuition, but i haven't thought it through
 
@Semiclassical oh. well, look up snake lemma.
 
@evinda Sounds like a good plan.
 
@TedShifrin yeah. stupid, really. and zariski topology is not really a good topology.
 
lol, that's what i was just looking up in another chrome tab
 
just a fancy language to handle affine schemes
and i am not really in the position to appreciate schemes yet
 
3:48 PM
schemes are one of those words i don't even try to understand
 
natural to look at level sets of functions as your closed sets, @Balarka
 
@BalarkaSen "good" is a relative term-some things are good for certain tasks
 
mostly b/c i can't see any reason i'd care about them from an applied math perspective
whereas stuff like homology matters to me b/c i care about integrals on riemann surfaces
 
3^{n} \equiv 3^{n \mod 10^k} (\mod 10^k)— how can I show that this is true for k>1?
 
sheaf cohomology, @Semiclassical
and sheaves are related to schemes
 
3:50 PM
yes, well, i haven't heard a good explanation for why i should care about sheaves either :P
 
great, now i am speaking about stuff i have no idea about.
 
natural from an analytic viewpoint in terms of gluing local solutions together ... what's the obstruction to their making a global one, @Semiclassical ?
 
i can well believe you'd want those concepts if you wanted to have a properly rigorous discussion. i just don't care about rigor all that much
(physicist)
 
but that's where monodromy comes from ... duh
 
@Ted I dropped atiyah-macdonald because it felt dry. but i will have to revise my algebra before getting into cohomology, and might as well study a bit of comalg too [i read a bit about the idea of cohomology from hatcher and they seem to be getting at something like graded rings from cup products]
 
3:52 PM
The closure of $(0, 1)$ would be $\mathbb{R}$ wouldn't it?
 
@Balarka: I've told you many times that some things should wait until you're 18.
 
under the zariski topology on $\mathbb{R}$
 
@TedShifrin i should wait until 18 before reading cohomology?!
that'd be too late, i have to study multivariable calculus too.
 
I'm a big believer that one should understand differential forms ... they're a very concrete version of cohomology.
 
eh. the way i know monodromy is from drawing cycles in the complex plane, analytically continuing some parameter through a cycle to deform branch cuts/poles, and then looking at how the cycles have changed
 
3:53 PM
@TedShifrin yes, and for that i need differential geometry. that's why i'd be studying mult calc.
 
just showed my class yesterday in essence why $H^1(\Bbb R^2-\{0\}) \cong \Bbb Z$.
 
@TedShifrin Because race cars?
 
And that must mean that $(0, 1) \cup (2, 3)$ isn't a connected subset of $\mathbb{R}$ then?
 
huh?
 
You know, "laps"
 
3:54 PM
oh
 
my sense is that physics people have some sense of differential forms just from their background in stuff like EM and thermodynamics. but we're not very precise about it, and so would probably have a lot of misconceptions
 
why isn't it connected, @user112495? Give me a separation.
 
poincare duality with the fact that R^2 - {0} def rets onto S^1.
easy.
waits for smacks
 
@DanielFischer How can we prove an invariant for a recursive algorithm?
 
3:55 PM
Poincaré duality only works on compact, oriented manifolds, @Balarka
 
lol ok
 
race cars do that, too, it's called "driving between the lines"
 
@TedShifrin Whoops. It just means I can't use this specific theorem to prove that it's connected.
 
@evinda Induction?
 
but, for example, $H^1(\Bbb R^2-\{0\}) \cong \Bbb Z$ in physics terms would probably be stated in terms of a current and the magnetic field/vector potential that can be created by it.
 
3:56 PM
I haven't been paying attention, @user112495. I don't know to which theorem you refer.
 
@TedShifrinOh wait, i misread it. They are connected
 
@Semiclassical oh? mild interest
 
yes, @Semiclassical, indeed. I showed them $d\theta$ and argued why it couldn't be exact.
 
right
 
We're just starting differential forms ... this is a freshman/sophomore class.
 
3:58 PM
now that i see so much connection to differential geometry, i wish i knew some analysis
anyway, back to interpreting homology geometrically
 
@TedShifrin If C and D are connected subspaces of a topological space T, and if $C \cap \bar D \neq \phi$, then $K = C \cup D$ is connected.

As the closure of $(0, 1)$ is $\mathbb{R}$, they have nonempty intersection, and so all necessary conditions hold.
 
It doesn't say anything about closures in your theorem, @user112495
 
one of my favorite uses of diff. forms, though, is in deriving maxwell identities in thermodynamics. usually one has to do it a bit tediously, but there's a cute way to get it directly
 
And you'd first have to prove that $(0,1)$ is connected ... maybe you've already done that.
 
@TedShifrin Sorry, I'm using the notation $\bar D$ to denote the closure of D.
 
4:00 PM
yes, but there wasn't any closure in your statement.
 
@TedShifrin Doesn't that follow as (0, 1) is an interval?
 
I took a year of thermo in college, @Semiclassical. Loved it. What are you calling Maxwell identities?
wrong topology @user112495
 
@TedShifrin Which topologies does that hold for then?
 
basically just what you get from taking mixed second partials of the various thermodynamic potentials
 
The subspace/order topology from the usual topology on $\Bbb R$, @user112495
right, @semiclassical, that's what I thought. But that's really circular reasoning. $d^2 = 0$ is equivalent to mixed partials being 0.
 
4:03 PM
well, that's not what i was using as 'derivation'
 
oh?
 
start from a statement of the thermodynamic identity as $dE=T\,dS-p\,dV$
 
Yes
 
if only we could make turning a continuous function-stupid calendar
 
@TedShifrin As we're under the zariski topology, I know that $(0, 1)$ is open, and that its closure is all of $\mathbb{R}$. How can I show it is connected?
 
4:05 PM
then take the exterior derivative of both sides and use $d^2=0$ to get $dT\wedge dS=dp\wedge dV$
 
No, @user112495, $(0,1)$ isn't open
Precisely, @semiclassical. You're using $d^2=0$.
 
if that's what you're getting at in terms of circularity, i don't disagree. it's a formal derivation
 
Yup.
It's the same proof the chemists give without mentioning differential forms and just manipulating differentials formally.
 
well, there's one further formal wrinkle
 
@TedShifrin But I thought that under the zariski topology on $\mathbb{R}$, a subset of $\mathbb{R}$ is closed iff it consists of finitely many points or if it is all of $\mathbb{R}$?
 
4:06 PM
what i really like is that you can get all four versions out of that one expression without referring to the potentials
 
Right, @user112495. So is the complement of $(0,1)$ a finite set?
 
@TedShifrin and $(0, 1)$ is neither of these.
 
namely, by expanding each of the four one-forms in terms of (say) $S$ and $V$, $S$ and $p$, etc.
 
@TedShifrin No, it's not.
 
AGH, @user112495. You are laboring under the misconception that any given set must be either open or closed. WRONG.
 
4:07 PM
including with the correct signs by taking account of the wedge product being antisymmetric
 
@Ted :P
 
yes, @Semiclassical, I know :)
 
heh, fair enough. i just like it as a way of quickly obtaining the correct expressions, which is a bit tedious otherwise
granted, yes, it's just formalism. but it's a useful formalism for a lazy physicist :P
 
@TedShifrin Oh, so it's not closed. And as it's complement isn't closed, it isn't open either.
 
right
 
4:11 PM
i suspect there are more full-blown applications of diff. forms in thermodynamics, say for how one understands stuff like heat capacities and cycles
but it's not something i've labored with myself. (i tend to use stat mech much more than thermo in the stuff i do)
 
@TedShifrin How do I go from here to determine whether or not it's connected?
 
show you can't separate?
back later
 
@user112495 What does a closed set look like in ZT?
 
@DavidWheeler It is either the whole set we are looking in, or it contains only finitely many points.
 
Ok, so suppose $(0,1) = U + V$, where $U,V$ are closed.
Wouldn't that mean $(0,1)$ was finite?
 
4:16 PM
ugh. i really should start recreating the mathematica stuff i was doing yesterday. computer crashes are such a pain when you're sloppy about saving :P
 
@DavidWheeler Oh, so we can't write (0, 1) as a union of two finite sets, as that will only have finitely many points. Thus either U or V must be $\mathbb{R}$. But this cannot be the case. So (0, 1) must not be connected.
@DavidWheeler So can I use the same argument to show that $(0, 1) \cup (2, 3)$ isn't connected?
 
In ZT, it's easier to work with closed sets.
 
How do you write in LaTeX the big intersection
 
\bigcap
 
@DanielFischer So is it right as follows?
So is it right as follows?

- Base Case: For $k=0$, the algorithm returns $1$, so it is correct since result=$n^{\lfloor \frac{k}{2} \rfloor}=n^0=1$.
- Induction Hypothesis: We suppose that $\forall 0 \leq j \leq k: result=n^{\lfloor{ \frac{j}{2} \rfloor }}$
- Induction Step: Suppose that the second argument of the algorithm is $k+1$.
Then result=pow($n, \lfloor \frac{k+1}{2} \rfloor$) $\overset{\text{ Induction hypothesis}}{=} n^{\lfloor \frac{k+1}{2} \rfloor}$
 
4:21 PM
@user112495 You've shown $(0,1)$ CANNOT be written as the disjoint union of 2 closed sets-that means it IS connected.
The same argument shows the union is also connected. It seems bizarre, because ZT is bizarre.
 
@DavidWheeler Thanks!
 
@user112495 Uh, wait a minute....
 
@DavidWheeler Wait, what?
 
We're still good on (0,1), but not on the union
Because for connectedness, we have to check in the relative ZT.
 
@DavidWheeler What do you mean by that?
 
4:28 PM
For example, $(-\infty,1)\cup(1,\infty)$ is open in ZT
So, in the relative ZT, we have for $A = (0,1) \cup (2,3)$, that $[(-\infty,1) \cup (1,\infty)] \cap A$ is open
 
@DavidWheeler Why do we need to check the relative ZT? I haven't come across this before.
 
Nevermind, I'm over-thinking it
 
@DavidWheeler So does the original argument work?
 
Yeah
Even in the relative topo, any open set intersect $A$, will still miss only finitely many points of either interval.
 
@Chris'ssis What are you doing now?
 
4:56 PM
@ABeautifulMind Just back from jogging (actually, very hard training). How about you?
@ABeautifulMind Well, I begin to feel more and more the mental stress, it affects my mood to a certain extent. I think I'll need a break from doing math, a longer one.
@robjohn didn't you start with the one with $\sin(2x)$?
 
@Chris'ssis I can see how to do that one, I think. It is just a sub and IBP
 
@robjohn You mean you have a solution or?
OK.
 
i still haven't done the triple integral one :/
right now i'm actually trying to figure out the simplest way to compute the following integral (which i know is a combination of complete elliptic integrals, but i'd appreciate a less tedious calculation)
 
@Semiclassical No hurry with that one.
 
$$\int_0^{2\pi} \sqrt{\frac{\lambda+e^{-i x}}{\lambda+e^{i x}}}\,dx$$
for $\lambda>0$
 
5:07 PM
@Chris'ssis I went out for a walk and haircut today.
 
@ABeautifulMind That's good. You arranged yourself to be beautiful, right? :-)
(maybe are you about to date a girl?:D)
 
@Chris'ssis I have made some plans to solve 99 per cent of my OCD in April. I hope I succeed. However, the other 1 per cent might take another year or so.
@Chris'ssis Nope, I won't date anyone until I am totally well. =)
 
@ABeautifulMind What's the plan? How would you do that?
 
@Chris'ssis Hard to describe here. It's a very long story. But I might upset a lot of people next month, but I think it will be fine.
 
Hi jasper @ABeautifulMind
Hi @Chris'ssis
 
5:11 PM
@SayanChattopadhyay Hi
 
hi @sayan
 
@SayanChattopadhyay Hi. You can buy books online from Amazon, either amazon.in in India or amazon.com in US to ship to India.
 
@Semiclassical Interesting integral.
 
Hi @Ramanewbie
 
@SayanChattopadhyay o/
 
5:13 PM
Thanks jasper
I didn't get u @Hippalectryon
 
i thought so too. mathematica can't compute it, amusingly, or at least not until one tosses out the odd part of it (one can integrate over $[-\pi,\pi]$ instead since the integrand is periodic)
 
@hippa how did you arrive so fast ??
 
@Ramanewbie teleportation
 
"Rarely if ever expressible as a ratio of integers." - what does this mean?
 
@hippa "Hey, people, you see he's mad !" lol
 
5:14 PM
@SayanChattopadhyay o/ = hello
 
@AGoogler Rarely rational. Pun intended.
 
irrational
 
lol
 
(and even when mathematica does give it in general, it's not in the simplest form possible. mathematica is kind of stupid when it comes to complete elliptic integrals)
 
5:16 PM
I feel I am too oldfashioned
 
doe "general discussion" include things completely unrelated to math?
*does
 
@AGoogler Yes.
 
@robjohn You mean you use sub one time and then IBP and you're done? If so, your way is far far more brilliant than mine. At any rate, I admit I might miss some obvious ways though.
 
great
So um , I wanna learn programming , where should i start? I've been suggested python
 
Python is good
Lua too
Basic, also. There are tons of Basics out there
 
5:20 PM
i learned a bit of basic in school
 
I found it!
73
Q: Math and mental fatigue

yoyosteinJust a soft-question that has been bugging me for a long time: How does one deal with mental fatigue when studying math? I am interested in Mathematics, but when studying say Galois Theory and Analysis intensely after around one and a half hours, my brain starts to get foggy and my mental condi...

 
ok so , how should i learn python? many people are suggesting to learn by doing , but what do i do?
 
@robjohn how do you deal with the metal fatigue from doing math? I don't afford to take breaks until I publish my book ...
 
@AGoogler You could follow some tutorials online
 
Actually it's a question for all: how do you deal with the metal fatigue from doing math?
 
5:22 PM
There are some good websites out there
@Chris'ssis Chocolate
 
@Chris'ssis you need to take breaks from anything taxing.
 
@Hippalectryon which do you recommend?
 
@robjohn Yeah, I think so.
@Hippalectryon hmmm, it doesn't work anymore :-(
 
is "too long for a comment" a valid reason for writing comments as answers?
 
@AGoogler It has been done before, but only if it's way over the limit. And do mention that it was intended to be a comment.
@AGoogler Have you ever leart programming before ? For other languages ?
 
5:25 PM
@Chris'ssis breaks let me climb out of ratholes into which I might be descending
 
@Hippalectryon a bit of c++ (old console version ) , some basic and very little javascript
 
@robjohn Realizing I'm very irritable, tired, sleepy, there must be something wrong. I think you're right.
 
@Chris'ssis sounds like time for a break and some food.
 
@Hippalectryon i'll try thanks
 
5:28 PM
@robjohn And, yes, I skipped many meals in the last period of time.
 
@Chris'ssis That can't do you any good
 
Hello people
Can anyone help me with a good book recommendation?
 
What book?
 
@Hippalectryon The brain is better irrigated with blood when you eat less, so the performance is better.
 
@ABeautifulMind I posted a question about it
 
5:34 PM
Is it possible to simplify \sum_{i=1}^{k} \binom{k}{i} \cdot (-2^{k-i})^{i} \cdot (2^k-1)^{k- i} any further?
 
@Chris'ssis But disrupting your eating cycle is bad
 
@Hippalectryon Yeap, I feel that now.
 
@Chris'ssis Don't pressure yourself so much. No mathematician changes her/his eating habits for math.
 
@AGoogler Thanks, that seems so.
@robjohn I'm referring to this one $$\int_0^{\pi/2}\frac{x\log{\sin{(x)}}}{\sin(2x)}\,dx$$
 
my immediate intuition for that is to try to replace $x$ with some useful function $f(sx)$, and characterize that integral as the first derivative of $f$ at 0
 
5:58 PM
Also note that
$$\int_0^{\pi/2}\frac{x\log{\sin{(x)}}}{\sin(2x)}\,dx=\int_0^{\pi/2}\frac{x\log{‌​\tan{(x)}}}{\tan(x)}\,dx$$
 
interesting. eyeballing that, it seems like that should amount to writing the second log as $\log \sin x-\log \cos x$ and doing an $x\to \pi/2-x$ reversal on the second term
except that mucks up the $x$ out in front. hrm.
 
Integrating by parts again here $$\int_0^{\pi/2}\frac{x\log{‌​\tan{(x)}}}{\tan(x)}\,dx$$ we get the way to go (all combined with beta function too).
 
ahh, neat
 
:-)
 
AvZ
6:32 PM
Finally hit 1k!
 
6:43 PM
@robjohn If you're on $W_{0}, W_{-1}$, or $W_{1}$ and you circle around $z = - \frac{1}{e}$ twice, do you end up where you started?
 
@RandomVariable: you may have seen this already, but this MO answer looks relevant (including sources)
4
A: Equation between the two branches of the lambert w function

Alexandre EremenkoYou should specify what do you mean by "equation". If you mean algebraic equation, then evidently there is none. Because an algebraic equation has finitely many solutions, and if there is an algebraic equation $F(W_0,W_{-1})=0$, analytic continuation will give you infinitely many solutions. This ...

 
@RandomVariable IF you start on $\mathrm{W}_{-1}$ near $-\frac1e$ and go counter clockwise around $-\frac1e$ you will pass across the branch cut to $\mathrm{W}_0$. You can then go counter clockwise around $-\frac1e$ a full rotation then pass through the branch cut to $\mathrm{W}_1$. Then you can go counter clockwise a half a rotation and cross the branch cut onto $\mathrm{W}_{-1}$.
So if you are close to $-\frac1e$, you can rotate counter-clockwise $\pi$ on $\mathrm{W}_{-1}$, $2\pi$ on $\mathrm{W}_0$, then $\pi$ on $\mathrm{W}_1$, then start over again.
 

« first day (1670 days earlier)      last day (3352 days later) »